There are two resistors connected in parallel: R1-43 Ohms and R2-43 Ohms.
Determine the equivalent resistance. Round your answer to 2 significant digits only. For example, if the answer is 65.4 Ohms write 65.

Answers

Answer 1

If There are two resistors connected in parallel: R1-43 Ohms and R2-43 Ohms,  then the equivalent resistance is 21.5Ω.

A resistor is a passive two-terminal electrical component used in circuits to implement electrical resistance. Resistors have a variety of purposes in electronic circuits, including lowering current flow, adjusting signal levels, dividing voltages, biassing active components, and terminating transmission lines. High-power resistors that can generate many watts of heat instead of electrical energy can be utilised as test loads for generators, power distribution systems, and motor controls. With temperature, time, or operating voltage changes, fixed resistors' resistances only slightly fluctuate. Variable resistors can be utilised as force sensors, heat sensors, light sensors, volume controls, lamp dimmers, humidity sensors, and chemical activity sensors.

The equivalent resistance in parallel combination is

R(p) = R₁R₂÷R₁+R₂

putting all values,

R(p) = 43×43÷(43+43)

R(p) = 21.5Ω

To know more about Resistor :

https://brainly.com/question/24297401

#SPJ1


Related Questions

Bending is a winter sport in which participants push a 15 kg rock across a horizontal snow patch. In 3.0 seconds, a bender accelerates a rock from rest to 4.0 m/s. What is the average power of the bender for accelerating the rock?

Answers

We can use the formula for average power:

P = W / t

where W is the work done and t is the time taken. The work done can be calculated using the formula:

W = (1/2) * m * v^2

where m is the mass of the rock and v is its final velocity. Substituting the given values:

W = (1/2) * 15 kg * (4.0 m/s)^2 = 120 J

Substituting the given value of time:

t = 3.0 s

Now we can calculate the average power:

P = W / t = 120 J / 3.0 s = 40 W

Therefore, the average power of the bender for accelerating the rock is 40 watts.

a block of mass m is at rest on the table .is it possible for this block at rest to have only a single force acting on it​

Answers

Answer:

In short no , there is always minimum 2 forces.

Explanation:

First force: Weight force ( The force of gravity on the object attracting the block of masse m to earth)

Second force: Normal reaction ( This force is perpendicular to the surface )

Answer:

no

Explanation:

if only single force acts on the block it will stay at rest and it will accelarate in direction of force

Select a character from the book and
choose two character traits that you
believe this character has. Describe
how the character has each of these
traits using information and
examples from your book that prove
that they do.

Answers

Within Chinua Achebe's novel entitled "Things Fall Apart", there lies a figure of paramount importance: Okonkwo.

How to explain the character

This individual is marked by two fundamental traits - determination and an unyielding dread of revealing his vulnerability, for he places immense value in tradition and rampant masculinity.

His ironclad willpower fuels his ambitions to attain respect and success within his community, through unwavering persistence and ceaseless diligence. Empowered by his fearsome strength and exceptional valor on the battlefield, along with his gathering wealth and spouses, this man gradually rises above his peers in stature, receiving adoration and honor in return.

Learn more about character on

https://brainly.com/question/898716

#SPJ1

The Children's Health Insurance Program (CHIP) is designed to:
A. provide low-cost health coverage to children in families that earn
too much for Medicaid.
B. enable families with children to compare the costs of health care
plans.
C. enable families to find medical professionals for their children
within their health care network.
D. provide low-cost health coverage to children in families that
receive Medicare.
SUBMIT

Answers

The Children's Health Insurance Program (CHIP) is designed to provide low-cost health coverage to children in families that receive Medicare.

Hence option D is correct.

The Children's Health Insurance Programme (CHIP) offers medical care to children under the age of 18 whose parents earn too much to qualify for Medicaid but not enough to pay for private coverage. CHIP was enacted by Congress during the Clinton administration in 1997.

The Children's Health Insurance Programme (CHIP) is a federal healthcare programme in the United States that is handled and designated differently by each state. For example, New York's programme is known as Child Health Plus, whereas Arkansas' programme is known as ARKids.

The federal government distributes matching funding to each state, similar to how Medicaid operates.

Hence option D is correct.

To know more about (CHIP) :

https://brainly.com/question/8395614

#SPJ1.

1. Determine the average of the three trials for each material.
Mystery A = ___30_______
Mystery B = ___2.8_______

In which material would light travel faster, Mystery A or Mystery B? Explain

2. As the index of refraction for the second medium is increased, what effect does this have on the angle of refraction? When it comes in at a lower angle, the ray bends more.

3. Write a conclusion for this lab.

Answers

The lab experiment found that light travels faster in Mystery A compared to Mystery B, with average speeds of 3.0 and 2.8, respectively. The increase in the index of refraction for the second medium led to a higher angle of refraction, resulting in light bending more. These findings have practical implications for optics and communications.

1. Light would travel faster in Mystery A since the average speed of light in Mystery A (3.0) is higher than Mystery B (2.8).

2. Increasing the index of refraction for the second medium leads to an increase in the angle of refraction. When light comes in at a lower angle, it bends more.

3. In conclusion, this lab experiment showed that the speed of light in a material is influenced by the material's index of refraction. Mystery A had a higher average speed of light compared to Mystery B, indicating that light travels faster in Mystery A. Additionally, the angle of refraction increased as the index of refraction for the second medium was increased. These findings have practical applications in the field of optics and communications.

Hence,The laboratory experiment discovered that, with average speeds of 3.0 and 2.8, respectively, light moves more quickly in Mystery A than Mystery B. Light bent more as a result of the second medium's increased index of refraction due to a higher angle of refraction. For optics and communications, these findings have real-world applications.

To learn more about Snell's Law of refraction click:

https://brainly.com/question/24321580

#SPJ1

A voltage of 65 V happen to a 2.53 nC. Calculate the work done.

Answers

The work done can be calculated using the formula:

W = Q * V

where W is the work done in joules (J), Q is the charge in coulombs (C), and V is the voltage in volts (V).

In this case, Q = 2.53 nC = 2.53 x 10^-9 C and V = 65 V.

Substituting these values into the formula, we get:

W = 2.53 x 10^-9 C * 65 V

W = 1.64 x 10^-7 J

Therefore, the work done is 1.64 x 10^-7 joules (J).

Two speakers create identical 240hz sound waves. a person is 1.47m from the speaker 1. What is the minimum distance to speaker 2 for there to be DESTRUCTIVE INTERFERENCE at that spot?

Answers

Two speakers create identical 240hz sound waves. a person is 1.47m from the speaker 1.
What is the minimum distance to speaker 2 for there to be
DESTRUCTIVE
INTERFERENCE at that spot?

1. Which of the following is an elementary particle?
a. quark
b. proton
c. neutron
d. atoms

Answers

Answer:

a

Explanation:

The correct answer is a. quark. Quarks are elementary particles that are the building blocks of protons and neutrons, which are composite particles made up of quarks and other particles. Protons, neutrons, and atoms are not elementary particles, as they are made up of smaller particles. Protons and neutrons are composed of quarks and other particles, while atoms are composed of protons, neutrons, and electrons.

A blue car of length 4.52 m is moving north on a roadway
that intersects another perpendicular roadway. The width of the intersection from near edge to far edge is 28.0 m. The blue car has a constant acceleration of magnitude 2.10 m/s2 directed south. The time interval required for the nose of the blue car to move from the near (south) edge of the intersection to the north edge of the intersection is 3.10 s. (a) How far is the nose of the blue car from the south edge of the intersection when it stops? (b) For what time interval is any part of the blue car within the boundaries of the intersection? (c) A red car is at rest on the perpendicular
intersecting roadway. As the nose of the blue car enters the intersection, the red car starts from rest and accelerates east at 5.60 m/s2. What is the minimum distance
from the near (west) edge of the intersection at which the nose of the red car can begin its motion if it is to enter the intersection after the blue car has entirely left the intersection? (d) If the red car begins its motion at the position given by the answer to part (c), with what speed does it enter the intersection?

Answers

The distance of the blue car from the edge of the intersection, when it stops, is 35.9 m, the time interval of the blue car within the boundaries of the intersection is 4.04 s, the minimum distance is 45.8 m, and the speed of the car is 22.6 m/s.

From the given,

A) the distance of the blue car from the south edge of the intersection when it stops =?

The width of the intersection = 28m

Acceleration = -2.10 m/s²

time interval = 3.10 s

By using the equation

x = x₀ + v₀t + 1/2 (at²)

28 = 0 + v₀(3.10) + 1/2 (-2.10 ×(3.10)²)

v₀ = 12.3 m/s

v² = v₀² + 2a (x-x₀)

(x-x₀) = Δx = v²-v₀² / 2a

  Δx  = 35. 9m

B) the time interval=?

distance covered by the blue car = 4.52 + 28 = 32.52 m

By using the relation,

x = x₀ + v₀t + 1/2 (at²)

32.52 = 0 + (12.3)t + 1/2 (-2.10)t²

-1.05t²+12.3t-32.52 = 0

This is the quadratic equation. By solving it, time t= 4.04s,7.66s. The desired time is t = 4.04 s, and the tail of the blue car leaves the intersection.

C) the minimum distance is=?

x = x₀ + v₀t + 1/2 (at²)

  = 0 + 0 + 1/2 (5.60 (4.04)²)

 = 45.8 m

The minimum distance of the blue car is 45.8m

D) speed of the car=?

the velocity equation

v = v₀ + at

= 0 + (5.60 ×4.04)

= 22.6 m/s

The velocity of the car is 22.6 m/s.

To learn more about the Equation of motion:

https://brainly.com/question/29278163

#SPJ1

3) Vector A is 2.8 cm at 60° above the positive x-axis. Vector B is 1.90 cm at 60 below the
positive x-axis. Use components to find the following
a) A+ B
b) A-B
c) B-A

Answers

The following vector components are:

a) A+ B - 84.21°

b) A-B - 18.74°

c) B-A - -84.21°

How to find vector components?

To use components to find the given vectors, break each vector into its x and y components using trigonometry.

Assume the positive x-axis points to the right and the positive y-axis points up.

For vector A, the magnitude of the x-component is given by Acos(60°) and the magnitude of the y-component is given by Asin(60°),

Ax = Acos(60°) = 2.8 cm × 0.5 = 1.4 cm

Ay = Asin(60°) = 2.8 cm × 0.866 = 2.425 cm

For vector B,  the negative sign since the vector is below the x-axis,

Bx = Bcos(60°) = 1.9 cm × 0.5 = 0.95 cm

By = -Bsin(60°) = -1.9 cm × 0.866 = -1.6494 cm

a) To find A + B, add the x and y components separately:

(A + B)x = Ax + Bx = 1.4 cm + 0.95 cm = 2.35 cm

(A + B)y = Ay + By = 2.425 cm - 1.6494 cm = 0.7756 cm

Then, the vector A + B has a magnitude of √[(2.35 cm)² + (0.7756 cm)²] = 2.466 cm and an angle of tan⁻¹(0.7756 cm / 2.35 cm) = 18.74° above the positive x-axis.

b) To find A - B, subtract the x and y components separately:

(A - B)x = Ax - Bx = 1.4 cm - 0.95 cm = 0.45 cm

(A - B)y = Ay - By = 2.425 cm + 1.6494 cm = 4.0744 cm

Then, the vector A - B has a magnitude of √[(0.45 cm)² + (4.0744 cm)²] = 4.091 cm and an angle of tan⁻¹(4.0744 cm / 0.45 cm) = 84.21° above the positive x-axis.

c) To find B - A, subtract the x and y components separately:

(B - A)x = Bx - Ax = 0.95 cm - 1.4 cm = -0.45 cm

(B - A)y = By - Ay = -1.6494 cm - 2.425 cm = -4.0744 cm

Then, the vector B - A has a magnitude of √[(-0.45 cm)² + (-4.0744 cm)²] = 4.091 cm and an angle of tan⁻¹(-4.0744 cm / -0.45 cm) = -84.21° below the positive x-axis.

Find out more on Vector here: https://brainly.com/question/27854247

#SPJ1

A pile driver of mass 300 kg is used to drive a pile of mass 500 kg vertically into the ground. The pile driver falls freely through a distance of 54.0 m, rebounding with a velocity relative to the pile and equal to the relative velocity immediately before impact. Determine: the velocity of the driver immediately before impact: the velocity of he pile immediately after the impact: the depth of penetration of the pile after impact given that the ground resisting force is constant and equal to 115 kN: (4 marks) the time taken for the penetration. (a) (b) (c) (d) (4 marks) (7 marks) (5 marks)​

Answers

a. By conservation of energy, the potential energy lost by the pile driver is equal to the kinetic energy gained by the pile and the pile driver. Therefore, (1/2) * 300 * v^2 = (1/2) * (300 + 500) * u^2, where v is the velocity of the pile driver immediately before impact and u is the velocity of the pile and pile driver immediately after impact. Solving for v, we get v ≈ 28.9 m/s.

b. By conservation of momentum, the momentum of the pile and the pile driver before impact is equal to the momentum of the pile and pile driver after impact. Therefore, 300 * v = (300 + 500) * u, where v is the velocity of the pile driver immediately before impact and u is the velocity of the pile and pile driver immediately after impact. Solving for u, we get u ≈ 17.3 m/s.

c. The work done by the ground resisting force is equal to the change in kinetic energy of the pile and pile driver. Therefore, (1/2) * (300 + 500) * u^2 = 115000 * d, where u is the velocity of the pile and pile driver immediately after impact, and d is the depth of penetration of the pile after impact. Solving for d, we get d ≈ 0.575 m.

d. The time taken for the penetration is equal to the distance penetrated divided by the average velocity during penetration. The average velocity during penetration is equal to u/2, since the velocity decreases linearly from u to 0 during penetration. Therefore, the time taken for the penetration is equal to d / (u/2) = 0.575 / (17.3/2) ≈ 0.066 s.
a. The velocity of the driver immediately before impact is given by:

v = sqrt(2gh) ≈ 34.14 m/s

where g is the acceleration due to gravity, h is the falling distance of the driver, and v is the velocity of the driver.

b. The velocity of the pile immediately after impact can be found by applying the law of conservation of momentum:

m1v1 + m2v2 = (m1 + m2)v'

where m1 and v1 are the mass and velocity of the driver before impact, m2 and v2 are the mass and velocity of the pile before impact, and v' is the velocity of the pile and driver immediately after impact.

Since the driver falls freely, its initial velocity is equal to v. The velocity of the pile before impact is zero, since it is stationary. Therefore:

(300 kg)(34.14 m/s) + (500 kg)(0 m/s) = (300 kg + 500 kg)v'

Solving for v', we get:

v' ≈ 20.48 m/s

c. The depth of penetration of the pile after impact can be found by applying the work-energy principle:

W = ΔK

where W is the work done by the ground resisting force, ΔK is the change in kinetic energy of the pile and driver, and K = (1/2)(m1 + m2)v'^2 is the final kinetic energy of the system.

The work done by the ground resisting force is:

W = Fd = (115 kN)(d)

where d is the depth of penetration of the pile into the ground.

The change in kinetic energy of the system is:

ΔK = (1/2)(m1 + m2)(v^2 - v'^2)

Substituting the given values, we get:

(115 kN)(d) = (1/2)(300 kg + 500 kg)(34.14^2 - 20.48^2)

Solving for d, we get:

d ≈ 0.728 m

Therefore, the depth of penetration of the pile after impact is about 0.728 m.

d. The time taken for the penetration can be found by applying the equation of motion:

d = (1/2)gt^2

where d is the depth of penetration, g is the acceleration due to gravity, and t is the time taken for the penetration.

Substituting the given values, we

A radio is rated as 50 W. Calculate the energy transferred in Joules by the radio when it has been switched on for 2 minutes?

Answers

The energy transferred in Joules by the radio when it has been switched on for 2 minutes would be 6000 Joules.

Energy transfer

Power is defined as the rate of energy transfer or the rate at which work is done, and is given by the equation:

Power = Energy transferred / Time

Rearranging the equation to solve for energy transferred, we get:

Energy transferred = Power x Time

We are given:

Power = 50 W

Time = 2 minutes = 120 seconds

Therefore, the energy transferred by the radio when it has been switched on for 2 minutes is:

Energy transferred = Power x Time = 50 W x 120 s = 6000 J

In other words, the energy transferred by the radio is 6000 Joules.

More on energy transfer can be found here: https://brainly.com/question/18649915

#SPJ1

Keyana and Sam are testing the law of conservation of energy. They use the same ball and release it from the same vertical height. Keyana is using a frictionless track, while Sam's track has friction. They discover Keyana's ball had more kinetic energy than Sam's when it reached the bottom. Which statement best explains why this happened if energy is conserved? Sam's ball lost mass as it traveled along the track. Sam's ball interacting with the track converted energy into heat. Keyana's ball was able to gain momentum. Keyana's ball had more potential energy.

Answers

The true statement is "Sam's ball interacting with the track converted energy into heat." The correct option is B.

The law of conservation of energy states that energy cannot be created or destroyed, only transferred or converted from one form to another. This means that the total amount of energy in a closed system remains constant.

The friction between Sam's ball and the track caused some of the energy to be lost as heat, while Keyana's ball experienced no such loss due to the absence of friction in her experiment. Therefore, Keyana's ball retained more of its initial potential energy as kinetic energy, resulting in a greater velocity and hence more kinetic energy at the bottom.

Option A (Sam's ball lost mass as it traveled along the track) is not true because it is not possible for the ball to lose mass during the experiment. The mass of the ball is a constant value and is not affected by the experiment.

Option C (Keyana's ball was able to gain momentum) is not the best explanation because momentum is not conserved in this scenario since external forces like friction are acting on the ball. The ball is only gaining kinetic energy.

Option D (Keyana's ball had more potential energy) is not true because both Keyana and Sam released the ball from the same vertical height. Therefore, both balls had the same initial potential energy. The difference in their kinetic energies at the bottom can be explained by the difference in their conservation of energy due to friction.

Therefore, The correct statement that best explains why Keyana's ball had more kinetic energy than Sam's when it reached the bottom, even though energy is conserved, is: Sam's ball interacting with the track converted energy into heat.

To learn more about the conservation of energy click:

brainly.com/question/2137260

#SPJ1

An acorn falls from rest from the top of a 19m tall oak tree. How long does it take for the acorn to fall to the ground? How fast is the acorn going before it hits the ground?

Answers

Answer:

We can solve this problem using the kinematic equation:

y = 1/2 * g * t^2

where y is the height of the tree, g is the acceleration due to gravity (9.8 m/s^2), and t is the time taken to fall to the ground.

We can solve for t using:

t = sqrt(2y/g)

Plugging in the values, we get:

t = sqrt(2(19)/9.8)

t = 2.19 seconds

So, it takes 2.19 seconds for the acorn to fall from the tree to the ground.

To find the velocity of the acorn just before it hits the ground, we can use:

v = g * t

Plugging in the values, we get:

v = 9.8 * 2.19

v = 21.46 m/s

So, the acorn is going approximately 21.46 m/s just before it hits the ground.

Explanation:

100 points & Brainliest
A ball of mass m = 0.275 kg swings in a vertical circular path tied to a string of L = 0.850 m long. (a) What are the forces acting on the ball at any point on the path? (b) Draw force diagrams for the ball when it is at the bottom of the circle and when it is at the top. (c) If its speed is 5.20 m/s at the top of the circle, what is the tension in the string there? (d) If the string breaks when its tension exceeds 22.5 N, what is the maximum speed the ball can have at the bottom before that happens?​

Answers

At any point on the circular path, there are two forces acting on the ball: the tension force T from the string and the gravitational force mg acting downward. Additionally, at the top of the circle, there is a normal force N acting upward to balance the weight of the ball.

At the top of the circle, the tension force T and the weight of the ball mg are both acting on the ball in the same direction, so the net force is:

F_net = T - mg

Since the ball is moving in a circle at a constant speed, the net force must be equal to the centripetal force:

F_c = mv^2 / r

where m is the mass of the ball, v is its speed, and r is the radius of the circle, which is equal to the length of the string L. Therefore:

T - mg = [tex]mv^2[/tex] / L

Solving for T, we get:

T = mg + [tex]mv^2[/tex] / L

Substituting the given values, we get:

T = (0.275 kg)(9.81 m/s^2) + (0.275 kg)(5.20 m/s)^2 / 0.850 m = 3.06 N

Therefore, the tension in the string at the top of the circle is 3.06 N.

At the bottom of the circle, the tension force T is pointing upward, opposing the gravitational force mg, which is pointing downward. The net force is:

F_net = T - mg

To find the maximum speed the ball can have at the bottom before the string breaks, we need to find the maximum tension that the string can withstand. From the given information, the maximum tension is 22.5 N.

Setting F_net equal to the maximum tension, we get:

T - mg = 22.5 N

Solving for v, we get:

v = sqrt[(T - 22.5 N) L / m]

Substituting the given values and solving for v, we get:

v = sqrt[(3.06 N - 22.5 N) (0.850 m) / 0.275 kg] = 3.89 m/s

Therefore, the maximum speed the ball can have at the bottom before the string breaks is 3.89 m/s.

For more details regarding force, visit:

https://brainly.com/question/13191643

#SPJ1

How to solve the question, pls ignore my answer ? I don't know how to finsih​

Answers

The final velocity of the puck, v, is determined as 3 m/s.

What is the impulse received by the puck?

The impulse received by the puck is calculated by applying the following formula.

impulse received = change in momentum of the puck = area under the curve

Area under the curve = area of triangle

Area of triangle = ¹/₂ x b x h

where;

b is the base = ( 5 ms - 2 ms) = 3 ms = 0.003 sh is the height = 160 N

Area = ¹/₂ x 0.003 s x 160 N

Area = 0.24 Ns

Therefore, impulse (J) = change in momentum (ΔP) = 0.24 Ns

The final velocity of the puck is calculated as follows;

m(vf - vi) = ΔP

where;

vf is the final velocity of the puckvi is the initial velocity of the puckm is the mass of the puck

Let vf be in positive direction,

then vi will in negative direction

0.03 kg(vf - (-5 m/s)) = 0.24 Ns

vf + 5 = 0.24/0.03

vf + 5 = 8

vf = 8 - 5

vf = 3 m/s

Learn more about impulse here: https://brainly.com/question/229647

#SPJ1

Jason's heartbeat is measured to be 65 beats per minute.
What is the frequency of heartbeats in hertz?
What is the period for each heartbeat in seconds?

Answers

Answer:

To find the frequency of heartbeats in hertz, we need to convert the beats per minute (BPM) to beats per second (BPS), since frequency is measured in hertz (Hz) which is equivalent to cycles per second.

Frequency = BPM / 60

Frequency = 65 / 60

Frequency ≈ 1.083 Hz

To find the period for each heartbeat in seconds, we need to find the reciprocal of the frequency, which gives the time duration for one complete cycle.

Period = 1 / Frequency

Period = 1 / 1.083

Period ≈ 0.922 seconds per beat

An axon is a

long, tubelike structure extending from a neuron's cell body

branch-like fiber extending in clusters from a neuron's cell body

neuron's cell body

messenger of the nervous system.

Answers

An axon is a long, tubelike structure extending from a neuron's cell body.

option A is the correct answer.

What is axon?

An axon is a long, tubelike structure extending from a neuron's cell body.

An axon is respnsible for transmitting nerve impulses, or action potentials, away from the cell body to other neurons, muscles, or glands.

Thus, an axon is a long, tubelike structure of a neuron that carries nerve impulses away from the cell body and towards other neurons, muscles, or glands. It is the primary means by which neurons transmit information throughout the nervous system.

Learn more about axon here: https://brainly.com/question/27274584

#SPJ1

An electron is accelerated by a constant electric field of magnitude 403 N/C.
(a) Find the acceleration of the electron.
(b) Find the electron's speed after 1.41 ✕ 10-8 s, assuming it starts from rest.

Answers

An electron is accelerated by a constant electric field of magnitude 403 N/C.  The acceleration of the electron is 7 × 10¹³ m/s². electron's speed after 1.41 ✕ 10-8 s is 987000 m/s.

Electric field is field around electrically charged particle where columbic force of attraction or repulsion can be experienced by other charged particles. It is denoted by letter E and it's SI unit is V/m Volt per meter or N/C newton per coulomb.

F = qE where E is electric field, q = 1.60 × 10⁻¹⁹ C is charge on the electron and F is Force on the electron

F = 1.60 × 10⁻¹⁹ C ×  403 N/C

F = 644.8  × 10⁻¹⁹ N

F = ma

where m =  9.1 × 10⁻³¹ kg , mass of the electron

a = F/m

a =  644.8  × 10⁻¹⁹ N ÷ 9.1 × 10⁻³¹ kg

a = 7 × 10¹³ m/s²

The acceleration of the electron is 7 × 10¹³ m/s².

The electron's speed after 1.41 ✕ 10⁻⁸ s is,

v = u + at

where u is initial velocity, which is zero.

v = at

v = 7 × 10¹³ m/s² × 1.41 ✕ 10⁻⁸ s

v = 987000 m/s

To know more about Electric field :

https://brainly.com/question/8971780

#SPJ1.

Two particles are fixed to an x axis: particle 1 of charge 91=2.03 x 108 C at x = 22.0 cm and particle 2 of charge 92=-4.0091 at x=71.0 cm. At what coordinate on the x axis is the electric field produced by the particles equal to zero?

Answers

The electric field produced by the two particles is equal to zero at a coordinate on the x-axis of 41.1 cm.

The electric field produced by the two particles can be found using the equation E = kq/r^2, where k is the Coulomb constant, q is the charge of the particle, and r is the distance from the particle. At a point where the electric fields produced by the two particles are equal in magnitude and opposite in direction, the net electric field will be zero.

Using this information, we can set the electric fields produced by each particle equal to each other and solve for the position where they cancel out. This gives us:

k(2.03 x 10⁸)/[(x - 0.22)²] = -k(4.0091)/[(x - 0.71)²]

Simplifying and solving for x gives:

x = 0.411 m or 41.1 cm

As a result, the electric field generated by the two particles is equal to zero at 41.1 cm on the x-axis.


To know more about the Particle, here

https://brainly.com/question/16792582

#SPJ1

In a place where the atmospheric pressure is patm=1atm and the acceleration of gravity g=10m/s² we carry out Toricelli's experiment with water instead of mercury. If the density of water is 1gr/m³ calculate the height h of the column of water in the tube​

Answers

If we carry out Torricelli's experiment with water instead of mercury then the density of water is 1gr/m³ the height h of the column of water in the tube​ is 10.3 m.

Glass tubes were brittle and difficult to come by at the time. When filled with a kilogramme of mercury, they frequently shattered. The experiment, however, was completed with the assistance of a professional helper. The mercury in the tube dropped and settled at 76 centimetres above the level in the dish. Torricelli was accurate in his assumption that the mercury ascended in the tube due to the weight of the atmosphere pressing down on the mercury in the dish, and that the space above the mercury column was a vacuum. It was the first time a hoover had been made and recognized in the laboratory.

In this problem, Both the pressure due to mercury and water is equal,

The normal atmospheric pressure is 76 cm of Hg,

ρgh' = σgh

where ρ and σ are densities of water and mercury resp. h' and h are heights of water and mercury resp.

ρh' = σh

h' = σh/ρ

h' = 13600 kg/m³× 0.76/997

h' = 10.3 m

To know more about Torricelli's law:

https://brainly.com/question/30479009

#SPJ1.

Calculate the energy changes corresponding to the transitions of the hydrogen atom. From n = 3 to n = ∞.

Answers

Answer: ΔE = -2.42 × 10^-19 J

Explanation:

The energy of an electron in the nth energy level of a hydrogen atom is given by the following formula:

E = (-2.18 × 10^-18 J) × (Z^2 / n^2)

where Z is the atomic number (1 for hydrogen) and n is the principal quantum number.

The energy change corresponding to a transition from energy level n1 to energy level n2 is given by the formula:

ΔE = E2 - E1 = (-2.18 × 10^-18 J) × Z^2 (1/n2^2 - 1/n1^2)

Given that the electron transitions from n = 3 to n = ∞, we can substitute n1 = 3 and n2 = ∞ in the above formula to obtain:

ΔE = (-2.18 × 10^-18 J) × 1^2 (1/∞^2 - 1/3^2)

ΔE = (-2.18 × 10^-18 J) × (1/9)

ΔE = -2.42 × 10^-19 J

Therefore, the energy change corresponding to the transition of the hydrogen atom from n = 3 to n = ∞ is -2.42 × 10^-19 J.

The energy change for the transition of a hydrogen atom from n = 3 to n = ∞ is 1.511 eV. This transition represents the electron moving to an energy level where it is essentially unbound from the nucleus, resulting in an energy increase.

The energy changes corresponding to the transitions of a hydrogen atom can be calculated using the formula for energy levels in hydrogen:

E = -13.6 eV * (Z² / n²)

Where:

E is the energy of the electron in electronvolts (eV).

Z is the atomic number, which is 1 for hydrogen.

n is the principal quantum number, representing the energy level.

Given the transition from n = 3 to n = ∞, we can calculate the energy change:

Calculate the initial energy (n = 3):

Einitial = -13.6 eV * (1² / 3²) = -13.6 eV * (1/9) = -1.511 eV

Calculate the final energy (n = ∞):

Efinal = -13.6 eV * (1² / ∞²)

In the final state, as n approaches infinity, the energy becomes zero.

Calculate the energy change (ΔE):

ΔE = Efinal - Einitial = 0 - (-1.511 eV) = 1.511 eV

So, the energy change corresponding to the transition of a hydrogen atom from n = 3 to n = ∞ is 1.511 electronvolts (eV).

For more such information on: transition

https://brainly.com/question/30771613

#SPJ2

In a futuristic scenario, you are assigned the mission of making an enemy satellite that is in a circular orbit around Earth inoperative. You know you cannot destroy the satellite, as it is well protected against attack, but you can try to knock it out of its orbit so it will fly away and never return. What is the minimum amount of work applied to the satellite that is required to accomplish that? The satellite's mass and altitude are 993 kg and 227 km. Earth's mass and radius are 5.98×10^24 kg and 6370 km.

Answers

The minimum amount of work required to make the enemy satellite inoperative and push it out of its circular orbit is 6.972 × 10^9 joules.

To calculate the minimum amount of work required to knock the satellite out of its circular orbit, we need to determine the change in kinetic energy required to change the satellite's velocity. This change in kinetic energy can be calculated using the conservation of energy, which states that the total energy in a closed system remains constant.

The kinetic energy of an object in motion can be expressed as:

K = (1/2)mv^2

Where:

K = Kinetic energy

m = Mass of the object

v = Velocity of the object

To determine the velocity of the satellite, we can use the following formula:

v = sqrt(GM/r)

Where:

G = Universal gravitational constant = 6.6743 × 10^-11 N m^2/kg^2

M = Mass of the Earth = 5.98×10^24 kg

r = Altitude of the satellite above the Earth's surface + radius of the Earth = 6,997 km

v = sqrt(6.6743 × 10^-11 × 5.98×10^24 / 6,997×10^3) = 7,650 m/s

To change the satellite's velocity, we need to calculate the new velocity required to push the satellite out of its circular orbit. We can use the following formula to calculate the escape velocity required to leave the Earth's gravitational field:

Ve = sqrt(2GM/r)

Ve = sqrt(2 × 6.6743 × 10^-11 × 5.98×10^24 / 6,997×10^3) = 11,186 m/s

To calculate the change in kinetic energy required to change the satellite's velocity from its initial velocity to the escape velocity, we can use the following formula:

ΔK = (1/2)m(Δv)^2

Where:

ΔK = Change in kinetic energy

m = Mass of the satellite

Δv = Change in velocity required to reach escape velocity = Ve - v

Δv = 11,186 m/s - 7,650 m/s = 3,536 m/s

ΔK = (1/2) × 993 kg × (3,536 m/s)^2 = 6.972 × 10^9 J

Therefore, The adversary spacecraft must be rendered inoperable and forced out of its elliptical orbit with a minimum of 6.972 × 10^9 joules of work.

To learn more about momentum and  impulse equations click:

brainly.com/question/30101966

#SPJ1

5. A body moving with uniform acceleration has a velocity 12 m/s in the positive x direction when its x coordinate is 3cm. If its x coordinate 2 s later is -5 cm, what is the magnitude of its acceleration? ​

Answers

The magnitude of acceleration can be calculated using the following kinematic equation:

x = x0 + v0t + 1/2at^2

where
x = final position = -5 cm
x0 = initial position = 3 cm
v0 = initial velocity = 12 m/s
t = time = 2 s

Converting all units to SI units, we get:

x = -0.05 m
x0 = 0.03 m
v0 = 12 m/s
t = 2 s

Substituting these values into the equation and solving for a, we get:

a = 2(x - x0 - v0t) / t^2
a = 2(-0.05 - 0.03 - 12(2)) / (2)^2
a = -12.5 m/s^2

Therefore, the magnitude of acceleration is 12.5 m/s^2.
The magnitude of the acceleration of the body is 2.35 m/s^2.

We can use the following kinematic equation to solve for the acceleration:

x = x0 + v0t + 1/2 at^2

where x is the final position, x0 is the initial position, v0 is the initial velocity, t is the time, and a is the acceleration.

Plugging in the given values, we get:

-0.05 m = 0.03 m + 12 m/s * 2 s + 1/2 * a * (2 s)^2

Simplifying and solving for a, we get:

a = (0.05 m - 0.72 m)/2 s^2 = -0.335 m/s^2

Since the acceleration is in the opposite direction of the initial velocity, we take the absolute value to get the magnitude of the acceleration:

|a| = 0.335 m/s^2 ≈ 2.35 m/s^2.

Questions (complete sentences)

1. Determine the average of the three trials for each material.
Mystery A = ___30_______
Mystery B = ___2.8_______

In which material would light travel faster, Mystery A or Mystery B? Explain

2. As the index of refraction for the second medium is increased, what effect does this have on the angle of refraction? When it comes in at a lower angle, the ray bends more.

3. Write a conclusion for this lab.

Answers

The average of the three trials for

Mystery A = 30 and for Mystery B = 2.8.

1. To determine which material would allow light to travel faster, we need to compare their respective indices of refraction. The index of refraction is defined as the ratio of the speed of light in a vacuum to the speed of light in the medium. A higher index of refraction means that light travels slower in that medium.

Therefore, if Mystery A has a lower index of refraction than Mystery B, then light would travel faster in Mystery A. Conversely, if Mystery B has a lower index of refraction than Mystery A, then light would travel faster in Mystery B.

2. As the index of refraction for the second medium is increased, the angle of refraction decreases. This is because the speed of light is slower in a medium with a higher index of refraction, causing it to bend more as it enters the medium.

The relationship between the angle of incidence, angle of refraction, and indices of refraction is described by Snell's law, which states that n1 sin(theta1) = n2 sin(theta2), where n1 and n2 are the indices of refraction of the two media, and theta1 and theta2 are the angles of incidence and refraction, respectively.

3. This lab explored the properties of light as it travels through different materials with varying indices of refraction. By measuring the angles of incidence and refraction, we were able to calculate the indices of refraction for two mystery materials. Through further analysis, we determined which material allowed light to travel faster. This lab helped us to better understand the behavior of light as it interacts with different materials, and reinforced the importance of the index of refraction in determining the speed of light in a given medium.

To know more about the Refraction, here

https://brainly.com/question/19132753

#SPJ1

Determine the magnitude of current in the 6 Ω resistor shown in Figure 3 if emf 1 has
an internal resistance of 1 Ω and emf 2 has an internal resistance of 0.2 Ω.

Answers

The magnitude of current in the 6Ω resistor is 0.72A.

EMF stands for electromotive force, which is the energy supplied by a source (such as a battery or generator) per unit of charge that passes through it, measured in volts. It represents the potential difference between the two terminals of the source when it is not connected to any circuit.

From the circuit diagram, we can see that the current flowing through the 6Ω resistor can be found by using Ohm's Law:

V = IR

where V is the voltage across the 6Ω resistor, I is the current flowing through it, and R is the resistance of the resistor.

To find V, we need to use Kirchhoff's Voltage Law (KVL) to determine the total voltage drop across the circuit. Starting from the top left corner and moving clockwise, we have:

V1 (emf) = IR1 + V2 (emf)

V2 (emf) - IR2 - IR1 = 0

Substituting V2 = -IR2 - 0.2I (since emf 2 has an internal resistance of 0.2Ω) into the first equation, we get:

V1 = I(R1 + R2 + 1) + 0.2I

Simplifying, we get:

V1 = I(7Ω) + 0.2I

Now we can solve for I:

I = V1 / (7Ω + 0.2Ω)

I = V1 / 7.2Ω

To find V1, we can use KVL again, starting from the bottom left corner and moving clockwise:

V1 - IR1 - V2 = 0

V1 - IR1 - (IR2 + 0.2I) = 0

Substituting V2 = -IR2 - 0.2I, we get:

V1 = I(R1 + R2 + 0.2) = I(7.2Ω)

Now we can substitute this expression for V1 into the equation for I:

I = (I(7.2Ω)) / (7Ω + 0.2Ω)

Simplifying, we get:

I = 0.72A

Therefore, the magnitude of current in the 6Ω resistor is 0.72A.

To learn more about Kirchhoff's rules click:

https://brainly.com/question/30201571

#SPJ1

An unhappy 0.400 kg rodent, moving on the end of a spring with force constant 3.50 N/m , is acted on by a damping force Fx=−bvx .

Answers

The equation of motion for the rodent is x(t) = -1.12cos(3.20t), and the damping force is Fd = -0.62*vx(t). The damping force will cause the amplitude of the motion to decrease over time, and the rodent will eventually come to rest at the equilibrium position.

We can use the following equations to solve this problem:

F = -kx (Hooke's Law)

F = ma (Newton's Second Law)

a = d^2x/dt^2 (Definition of Acceleration)

Fd = -bv (Definition of Damping Force)

x(t) = A*cos(ωt + φ) (Equation of Motion for Simple Harmonic Motion)

We will need to use these equations to find the displacement, velocity, and acceleration of the rodent as a function of time, and then use that information to calculate the damping force and solve for the parameters of the motion.

First, let's find the natural frequency of the system:

ω = sqrt(k/m) = sqrt(3.50 N/m / 0.400 kg) = 3.20 rad/s

Next, let's assume that the rodent starts at its maximum displacement and moves in simple harmonic motion. We can use the equation of motion for simple harmonic motion to write:

x(t) = A*cos(ωt + φ)

where A is the amplitude of the motion and φ is the phase angle.

To find A and φ, we need to use the initial conditions. We know that at t=0, the rodent is at its maximum displacement, so x(0) = A. We also know that at t=0, the velocity of the rodent is zero, so vx(0) = -Aωsin(φ) = 0. This means that either A=0 (the rodent is not moving) or sin(φ) = 0 (the rodent is moving with maximum velocity). We will assume that the latter is true, so sin(φ) = 0 and cos(φ) = 1.

Now we can write:

x(t) = A*cos(ωt)

To find A, we use the fact that the rodent has a mass of 0.400 kg and is moving on a spring with force constant 3.50 N/m. The force on the rodent is given by:

F = -kx = -3.50 N/m * A*cos(ωt)

At maximum displacement, the force is equal to the weight of the rodent:

mg = 0.400 kg * 9.81 m/s^2 = 3.92 N

So we can write:

3.92 N = -3.50 N/m * A

A = -1.12 m

Therefore, the equation of motion for the rodent is:

x(t) = -1.12cos(3.20t)

To find the velocity and acceleration of the rodent, we take the derivative of the displacement with respect to time:

vx(t) = dx/dt = 3.58sin(3.20t)

ax(t) = d^2x/dt^2 = -11.46cos(3.20t)

To find the damping force, we use the equation:

Fd = -bv = -bdx/dt = -b3.58sin(3.20t)

We don't know the value of b, so we can't solve for it directly. However, we can use the fact that the damping force is equal to the work done by the damping force over one cycle of motion. This work is equal to the energy lost by the system due to damping. Since the system is losing energy at a rate proportional to its velocity, we can write:

Energy lost per cycle = Average damping force * Distance traveled per cycle

The distance traveled per cycle is equal to 2piA = 7.04 m, since the rodent moves from its maximum displacement to its minimum displacement and back again in one cycle.

The average damping force over one cycle is equal to the time average of the damping force:

<Fd> = (1/T)∫[0,T] -bdx/dt dt

where T = 2*pi/ω is the period of the motion. Evaluating the integral gives:

<Fd> = (1/T)∫[0,T] -b(-1.12)3.20sin(3.20*t) dt

<Fd> = 3.58*b

Since the energy lost per cycle is also equal to (1/2)kA^2, we can write:

(1/2)kA^2 = <Fd>2pi*A

Solving for b, we get:

b = (kA)/(2pi)

Substituting the given values, we get:

b = (3.50 N/m * 1.12 m)/(2*pi) = 0.62 Ns/m

Therefore, the equation of motion for the rodent is:

x(t) = -1.12cos(3.20t)

vx(t) = 3.58sin(3.20t)

ax(t) = -11.46cos(3.20t)

and the damping force is given by:

Fd = -0.62*vx(t)

Note that the negative sign indicates that the damping force acts in the opposite direction to the velocity of the rodent. This means that the damping force will cause the amplitude of the motion to decrease over time, and the rodent will eventually come to rest at the equilibrium position.

Therefore,The equation of motion for the rodent is x(t) = -1.12cos(3.20t), and the damping force is Fd = -0.62*vx(t).

To learn more about Newton's law of motion click:

brainly.com/question/29775827

#SPJ1

3. Which energy resource is used to generate electricity without using any moving parts? A. geothermal B. hydroelectric C. nuclear D. solar​

Answers

Answer: Solar

Explanation: This is simple enough the only one of these options that do not require a pump or power plant is solar which is usually attained with a non-moving solar panel

hope this helps :)

D. Solar…………….hope this helps

Which of the following has the stages of change in the correct order?
A. Contemplation â€" Preparation - Pre-contemplation â€" Action â€"
Maintenance
B. Pre-contemplation â€" Contemplation â€" Preparation â€" Action
â€" Maintenance
C. Preparation â€" Pre-contemplation â€" Contemplation â€" Action
â€" Maintenance
D. Action â€" Contemplation â€" Preparation â€" Pre-contemplation -
Maintenance

Answers

Pre-contemplation â€" Contemplation â€" Preparation â€" Actionâ€" Maintenance has the stages of change in the correct order. Hence option B is correct.

In a religious setting, contemplation seeks a direct perception of the divine that transcends the mind, frequently in conjunction with prayer or meditation.

The term contemplation is derived from the Latin word contemplatio, which is derived from the Latin word templum, which is a plot of land dedicated for the taking of auspices or a place of worship. The latter originates from the Proto-Indo-European root *tem- ("to cut"), referring to a "reserved or cut out" area in front of an altar, or from the root *temp- ("to stretch, string"), alluding to a cleared (measured) space in front of an altar.[The Greek term (thera) was translated using the Latin word contemplatio.

Maintenance technically refers to functioning inspections, maintaining, repairing, or replacing required devices, equipment, machinery, building infrastructure, and supporting utilities in industrial, commercial, and residential settings. This has evolved over time to include a variety of wordings that express various cost-effective practises to maintain equipment operating; these operations take place either before or after a breakdown.

Hence option B is correct.

To know more about Maintenance :

https://brainly.com/question/29760355

#SPJ1.

A point charge of 1.0 C is 15 m from a second point charge, and the electric force on one of them due to the other is 1.0 N. What is the modulus of the second load? (k = 1/4πε0 = 8.99 × 109N∙m2/C2)

Answers

The modulus (or magnitude) of the second charge is approximately 3.34 × 10⁻⁶ C.

We can use Coulomb's law to solve this problem:

F = k * (q₁ * q₂) / r²

where F is the electric force between the two charges, q₁ and q₂ are the magnitudes of the charges, r is the distance between the charges, and k is the Coulomb constant.

We know that the electric force between the two charges is 1.0 N, that one of the charges has a magnitude of 1.0 C, and that the distance between the charges is 15 m. Therefore, we can solve for the magnitude of the second charge:

1.0 N = (8.99 × 10⁹ N∙m²/C²) * (1.0 C) * q₂ / (15 m)²

Solving for q₂, we get:

q₂ = (1.0 N) * (15 m)² / (8.99 × 10⁹ N∙m²/C²) ≈ 3.34 × 10⁻⁶ C

To know more about charge, here

brainly.com/question/3412043

#SPJ1

Other Questions
what is the difference between obeying out of guilt and obeying out of gratitude? There's no point ______ trying to get an interview with him. He never does interviews! A. for. B. about. C. over. D. in Please anwser this is all the points I have :) A metal ring lies on a table. The s pole of a bar magnet moves down toward the ring from above and per pendicular to its surface. Which an- swer and explanation correctly pre- dict the direction of the induced cur- rent as seen from above? a. Clockwise because the B field is down and increasing b. Clockwise because the B field is up and increasing c. Counterclockwise because the B field is down and ncreasing d. Counterclockwise because the B field is up and increasing. e. There is no current; it only changes when the N pole approaches a conductor consists of a circular loop of radius r and two long, straight sections. the wire lies in the plane of the paper and carries a current i. a) what is the direction of the magnetic field at the center of the loop? b) find an expression for the magnitude of the magnetic field at the center of the loop. 4. a long, straight wire carries a current i. a right-angle bend is made in the middle of the wire. the bend forms an arc of a circle of radius r. determine the magnetic field at point p, the center of the arc. 5. two parallel wires are separated by 6.00 cm, each carrying 3.00 a of current in the same direction. a) what is the magnitude of the force per unit length between the wires? b) is the force attractive or repulsive? 6. two parallel wires separated by 4.00 cm repel each other with a force per unit length of 2.00x104 n/m. the current in one wire is 5.00 a. a) find the current in the other wire. b) are the currents in the same direction or in opposite directions? c) what would happen if the direction of one current were reversed and doubled? please help me with this Compose a letter to a business of your choice to inquire about a specific job opportunity. State your desire to secure aposition with the company, then present reasons you want to join the company, as well as the skills and qualifications youpossess that make you an ideal candidate for the job. Your letter should use appropriate business letter formatting andmaintain a formal and professional tone.Learn about how to format a business letter here. How did Mrs. Linde support herself after her husband died? Which function represents the graph of f(x) = -4 lxl after it is translated 3 units down?A.g(x) = -4 lxl + 3B. g(x) = -4 lxl - 3C. g(x) = -4 lx + 3|D. g(x) = -4 lx - 3l Rewrite the following in the form log(c). log(5) + log(2) IM GIVING 50 POINTS!A box contains 1 plain pencil and 3 pens. A second box contains 5 color pencils and 5 crayons. One item from each box is chosen at random. What is the probability that a pen from the first box and a crayon from the second box are selected. Write your answer as a fraction in the simplest form Thanks to an initiative to recruit top students, an administrator at a college claims that this year's enteringclass must have a greater mean IQ score than that of entering classes from previous years. Theadministrator tests a random sample of 17 of this year's entering students and finds that their mean IQ scoreis 114, with a standard deviation of 15. The college records indicate that the mean IQ score for enteringstudents from previous years is 113.Is there enough evidence to conclude, at the 0.05 level of significance, that the population mean IQ score, ,of this year's class is greater than that of previous years? To answer, assume that the IQ scores of this year'sentering class are approximately normally distributed.Perform a one-tailed test. Then complete the parts below.(If Rococcan consult a list of Traditional project management approaches included multiple KPIs that predicted the success of a project. How do we demonstrate success of an Agile project? consider an industry that is made up of nine firms each with a marketshare (percent of sales) as follows firm a 35% firm b 20% firms c and d15% what is the four firm concentration ratio dim light at night make fat mice a study was conducted in which mice that had a dim light on at night 95% confidence interval which of the following is not related to a higher rate of recovery from schizophrenic disorders?question 4 options:sudden onsethealthy family situationlater onsethigher proportion of negative symptoms Given X = [b a / 4 a] , Y = [c d / a b] , and Z= [a c / 16 b] What is Y if X-2Y=Z?[2 -4/ -6 -2] [4 -8/ -12 -4] [-4 8/ 12 4] [12 0/ 12 -12] based on the average predictions of 60 economists, the u.s. gross domestic product (gdp) will expand by 2.7% this year. suppose the sample standard deviation of their predictions was 1%. at the 1% significance level, test if the mean forecast gdp of all economists is less than 3%. (you may find it useful to reference the t table.) a. Select the null and the alternative hypotheses.H0: = 3; HA: 3H0: 3; HA: > 3H0: 3; HA: < 3b. Calculate the value of test statistic. (Round final answer to 4 decimal places.)c. Find the p-value.0.05 p-value < 0.10p-value 0.10p-value < 0.010.01 p-value < 0.0250.025 p-value < 0.05d. At the 1% significance level, can we conclude that the mean forecast GDP of all economists is less than 3%?Yes, since we reject H0.Yes, since we do not reject H0.No, since we reject H0.No, since we do not reject H0. What characteristic of life? The body temperature of a dog is more stable than the temperature of its environment.a) sensitivityb) homeostasisc) ordered complexityd) reproductione) energy utilizationf) evolution In which of the following parts of a hair would a forensic scientist hope to find the best sample of mitochondrial DNA? Shaft Follicle Tip Cuticle